期權

術語在哪裡CCgamma從測量移動時來自問ñ問ñmathbb Q^{N}至問米問米mathbb Q^{M}?

  • February 1, 2022

考慮兩個措施 $ \mathbb Q^{M} $ 和 $ \mathbb Q^{N} $ ,以及兩筆現金 $ M $ 和 $ N $ , 進一步假設 $ X\frac{N}{M} $ 是一個 $ \mathbb Q^{M} $ -鞅。此外,協變 $ X $ 和 $ \frac{M}{N} $ 滿足以下條件:

$$ dX(t)\cdot d\frac{M(t)}{N(t)} = \frac{M(t)}{N(t)}X(t)\gamma(t)dt $$

最後我們假設:

$$ dX(t) = X(t)(\mu(t)dt+\sigma(t)dW(t)); ; ; \text{under }\mathbb Q^{M}, $$

然後指出,通過使用 Girsanov,它遵循

$$ dX(t) = X(t)(\mu(t)+\gamma(t) )dt+\sigma(t)dW(t)); ; ; \text{under }\mathbb Q^{N} $$

術語在哪裡 $ \gamma $ 從測量移動時來自 $ \mathbb Q^{N} $ 至 $ \mathbb Q^{M} $ ?

我的嘗試:由於使用 Girsanov 進行了論證,因此我試圖將其與我對 Girsanov 的了解相協調。

我們一般認為 $ \mathbb E^{\mathbb Q^{N}}[X]= \mathbb E^{\mathbb Q^{M}}[X\frac{M}{N}] $ , IE $ \frac{d\mathbb Q^{N}}{d\mathbb Q^{M}}=\frac{M}{N} $

我們怎麼寫$$ \frac{M(t)}{N(t)} $$但是以指數形式?

關於我所缺少的任何想法?

根據你的假設 $$ \begin{align*} dX(t)\cdot d\frac{M(t)}{N(t)} &= \frac{M(t)}{N(t)}X(t)\gamma(t)dt,\ dX(t) &= X(t)\big(\mu(t)dt+\sigma(t)dW(t)\big), \end{align*} $$ 在下面 $ \mathbb Q^{M} $ , 我們可以推測 $$ \begin{align*} d\frac{N(t)}{M(t)} = -\frac{N(t)}{M(t)}\frac{\gamma(t)}{\sigma(t)}dW(t) \tag{1} \end{align*} $$ 作為 $ \frac{N(t)}{M(t)} $ 應該是鞅 $ \mathbb Q^{M} $ 與計價器 $ M $ . 那麼,下 $ \mathbb Q^{M} $ , $$ \begin{align*} d\frac{M(t)}{N(t)} = \frac{M(t)}{N(t)}\bigg(\frac{\gamma^2(t)}{\sigma^2(t)} dt + \frac{\gamma(t)}{\sigma(t)} dW_t\bigg).\tag{2} \end{align*} $$

但是,我懷疑你的意思是 $$ \begin{align*} dX(t)\cdot d\frac{N(t)}{M(t)} &= \frac{N(t)}{M(t)}X(t)\gamma(t)dt. \end{align*} $$

從 $ (1) $ , $$ \begin{align*} \frac{N(t)}{M(t)} = \frac{N(0)}{M(0)} \exp\bigg(-\frac{1}{2}\int_0^t\frac{\gamma^2(s)}{\sigma^2(s)} ds - \int_0^t \frac{\gamma(s)}{\sigma(s)}dW(s) \bigg). \end{align*} $$ 基於 Girsanov 定理, $ \hat{W}={\hat{W}(t), , t \ge 0} $ , 其中, 對於 $ t \ge 0 $ , $$ \begin{align*} \hat{W}(t) = W(t) + \int_0^t \frac{\gamma(s)}{\sigma(s)}ds, \end{align*} $$ 是一個標準的布朗運動 $ \mathbb Q^{N} $ . 然後, $$ \begin{align*} dX(t) = X(t)\Big[\big(\mu(t)-\gamma(t) \big)dt+\sigma(t)d\hat{W}(t)\Big], \end{align*} $$ 在下面 $ \mathbb Q^{N} $ .

引用自:https://quant.stackexchange.com/questions/69675